Chap 4 - CVP Analysis

March 21, 2018 | Author: zyra liam styles | Category: Economic Institutions, Corporate Jargon, Pricing, Economics, Microeconomics


Comments



Description

Chapter 4Marginal costing and CVP analysis 90 MULTIPLE CHOICE Basic concepts 1. Cost-volume-profit analysis assumes that over the relevant range A. Variable costs are nonlinear. C. Selling prices are unchanged. B. Fixed costs are nonlinear. D. Total costs are unchanged. (aicpa) 1. C ? A basic assumption within the relevant range.  Relevant range is a band (or stretch) of activity where the behavior of sales and costs is predictable. The basic assumptions used in management accounting within the relevant range are: a. Costs behavior - costs are segregated as to fixed or variable in behavior. b. Linearity - the behavior of total fixed cost, total variable cost, total costs, and total sales is linear (i.e., straight line). c. Fixed costs - total fixed cost does not change, while unit fixed cost changes; that is, UFC increases as production decreases and it decreases as production increases. d. Variable costs - total variable cost changes in direct relation to change in the volume of production and sales, while unit variable cost is constant. Also, total variable costs are not affected by the change in unit sales price. e. Unit sales price - unit sales price is constant. f. Sales mix - the company produces only one product, or in multi-product operations, the sales mix is constant. g. WIP - there is no work-in-process inventory. h. Production equals sales - there is no change in the number of units in finished goods inventory; meaning, production equals sales. Choice-letter “c” is correct, sales price is assumed to be constant within the relevant range. Choice-letters “a” and “b” are incorrect because variable costs and fixed costs are assumed to be linear. Choice-letter “d” is incorrect because total costs change on account of the total variable costs. 2. Cost-volume-profit analysis assumes that over the relevant range total. A. Revenues are linear. C. Variable costs are nonlinear. B. Costs are unchanged. D. Fixed costs are nonlinear. (aicpa) 2. A ? Basic assumptions within the relevant range.  Choice-letter “a” is correct because the linearity of revenues and costs is one of the basic assumptions in the CVP analysis. Choice-letter “b” is incorrect because total costs would change because of the change in variable costs. Choice-letters “c” and “d” are incorrect because variable costs and fixed costs are assumed to be linear. 3. Breakeven analysis assumes linearity over the relevant range with respect to Total costs Total revenue Total costs Total revenue A. Yes No C. No Yes Chapter 4 B. Yes (aicpa) Marginal costing and CVP analysis Yes D. No 91 No 3. B ? Breakeven linearity assumptions over the relevant range.  Relevant range is a band or width of activity (e.g., in terms of levels of output or the unit of measurement used) where the sales and costs could be predicted with reasonable certainty. Within this range, sales and total costs are assumed to behave linearly. Unit sales price is constant and total sales increase in direct relation with changes in production levels. Total cost is composed of fixed cost and variable cost where total fixed cost is constant regardless of changes in the levels of production and sales and unit variable cost is constant. Total variable costs change in direct relation with the changes in the unit of production levels. Choice-letter “b” is correct because total costs and total revenue are assumed to be linear within the relevant range. 4. Break-even analysis assumes that over the relevant range. A. Selling prices are unchanged. C. Total costs are unchanged. B. Variable costs are nonlinear. D. Fixed costs are nonlinear. (rpcpa) 4. A ? An assumption about breakeven analysis over the relevant range.  Within the relevant range, the following are the assumptions on cost-volume-profit analysis and breakeven analysis: the behavior of sales and costs is linear, total fixed costs is constant but unit fixed costs changes, total variable costs change but unit variable cost is constant; units sales price is constant; there is no change in the finished goods inventory (i.e., production equals sales); there is no work-in-process inventory; and the sales mix is constant. Within the relevant range, variable cost and fixed costs are linear and total costs change due to the change in variable costs. (Choice-letter “a” is correct). 5. The amount of variable cost per unit and total fixed cost within a relevant range behave this way in relation to production level: Production increases, unit variable cost increases, total fixed cost increases. Production decreases, unit variable cost decreases, total fixed cost decreases. Production increases, unit variable cost remains constant, total fixed cost remains the same. Production increases, unit variable cost decreases, total fixed cost remains the same. (rpcpa) 5. C ? The behavior of variable cost per unit and total fixed costs within the relevant range.  Total fixed costs remain constant within the relevant range. However, unit fixed cost decreases as production increases, and it increases as production decreases. Total variable cost changes in direct relation to the change in the level of production and sales within the relevant range. However, unit variable cost is constant within the relevant range. A. B. C. D. Chapter 4 Marginal costing and CVP analysis 92 Therefore, regardless of the change in the level of production, the total fixed costs and unit variable will be constant within the relevant range. Choice-letter “c” is the correct answer. 6. Assuming that a flexible budget is in use, production levels are expected to increase within a relevant ranged, the expected effect on fixed cost per unit (FCU) and variable costs per unit (VCU) would be A. FCU to decrease and VCU to decrease. B. FCU to decrease and VCU no change. C. FCU no change and VCU no change. D. FCU no change and VCU to decrease. (rpcpa) 6. B ? Using the flexible budget and assuming production increases within a relevant range, determine the expected effect on fixed costs per unit (FCU) and variable cost per unit (VCU) by an increase in production levels.  Within the relevant range, total fixed cost remains constant but FCU changes (that is, FCU decreases as production increases and FCU increases as production decreases). Also within the relevant range, total variable costs changes but VCU is constant. Choice-letter “b” is correct, FCU decreases, VCU does not change. Choice-letter “b” is correct. 7. One of the major assumptions limiting to reliability of break-even analysis is that A. The cost of productivity will continually increase. B. The cost of production factors varies with changes in technology. C. Total variable cost will remain unchanged over the relevant range. D. Total fixed cost will remain unchanged over the relevant range. (rpcpa) 7. D ? A major assumption limiting the reliability of breakeven analysis.  The breakeven analysis is based on the following assumptions within the relevant range: a. The behavior of sales and costs is linear within the relevant range. b. Total fixed costs are constant but unit fixed cost changes. c. Total variable costs change but unit variable cost is constant. d. Unit sales price is constant. e. Sales equal production; there is no change in the finished goods inventory. f. There is no work-in-process inventory. g. The sales mix ratio is constant. Choice-letter “d” is correct (refer to assumption letter ”b”), Choice-letter “a” is incorrect because efficiency and productivity have no direct relations with the BEP analysis; choice-letter “b” is incorrect because it is not an assumption in the BEP analysis, although the statement is true; choice-letter “c” is incorrect because total variable costs change (not unchanged) within the relevant range 8. At the breakeven point, the contribution margin equals total A. Variable costs. C. Selling and administrative costs. Chapter 4 B. Marginal costing and CVP analysis Sales revenues. (aicpa) D. Fixed costs. 93 8. D ? The amount of contribution margin at breakeven point.  At breakeven point, there is no profit or loss, meaning, total sales equal total costs. Alternatively, profit is contribution margin less fixed costs and expenses. Therefore, at breakeven point contribution margin equals total fixed costs. Choice-letter “d” is correct. 9. At breakeven point, fixed cost is always A. Less than contribution margin. C. More than variable cost. B. Equal to contribution margin. D. More than the contribution margin. (rpcpa) 9. B ? Fixed cost at breakeven point.  At breakeven point, total fixed cost equals contribution margin. This is so because, if fixed cost and contribution are of the same amount, there is no profit or loss. 10. An assembly plant accumulates its variable and fixed manufacturing overhead costs in a single cost pool, which is then applied to work-in-process using a single application base. The assembly plant management wants to estimate the magnitude of the total manufacturing overhead costs for different volume levels of the application activity base using a flexible budget formula. If there is an increase in the application activity base that is within the relevant range of activity for the assembly plant, which one of the following relationship regarding variable and fixed costs is correct? A. The variable cost per unit is constant, and the total fixed costs decrease. B. The variable cost per unit is constant, and the total fixed costs increase. C. The variable cost per unit and the total fixed costs remain constant. D. The variable cost per unit increases, and the total fixed costs remain constant. (cia) 10. C ? Relationship regarding variable and fixed costs within the relevant range.  Within the relevant range, unit variable cost and total fixed costs are constant. Choice-letter “c” is correct. 11. Contribution margin is the excess of revenues over A. Direct cost. C. Cost of good sold. B. Manufacturing cost. D. All variable costs. (rpcpa) 11. D ? A procedure in computing contribution margin.  The traditional way to compute contribution margin is by getting the difference between net sales and variable costs and expenses. Other procedures to determine contribution margin (CM) are as follows: CM = Quantity sold x Unit contribution margin income increases by every peso increase in contribution margin. however. Gross margin per unit for each additional unit sold. C. 12. The excess of contribution over fixed costs is profit. Once the breakeven point has been reached. When an organization is operating above the breakeven point. C. Relevant variable costs. D. . which is the unit contribution margin. B ? Effect to operating income. unit variable cost. Choice-letter “b” is also incorrect because the difference between revenues and manufacturing cost is manufacturing margin or manufacturing income.  Choice-letter “d” is correct. the relationship between two variables in a graph is linear. D ? The factor to be considered in analyzing curvilinear relationships. and total fixed cost are constant. 13. B. B. marketing strategy. meaning. Sales price unit for each additional unit sold. A relevant range of volume. operating income will increase by the A. Choice-letter “c” is also incorrect because the difference between net sales and cost of goods sold is gross profit. total sales are greater than total costs. Also. once the breakeven point has been reached. Choice-letter “b” is the right choice.Chapter 4 Marginal costing and CVP analysis 94 CM = Net sales x Contribution margin ratio CM = Fixed costs + Income before income tax Choice-letter “a” is incorrect because the difference between net sales and direct cost is direct margin or segment margin. Fixed and semi-variable costs.  Beyond the breakeven point. Choice-letter “a” is incorrect because semi-variable costs are normally curvilinear in its representation. In the short-term. D. including choice of product lines. This short-term range is otherwise known as relevant range and the linearity assumption is valid within the relevant range. It is a range of production activity where the behavior of costs and sales is linear. Choice-letters “b” and “c” are incorrect because other relevant costs are also included in the CVP analysis. 14. (aicpa) 13. Residual income rate. Contribution margin per unit for each additional unit sold. Margin of safety. (cma) 12. After breakeven point. A calculation used in a CVP analysis is the breakeven point. Curvilinear relationships exist over the long run. Relevant fixed costs. pricing of products. Cost-volume-profit analysis is a key factor in many decisions. contribution margin is greater than fixed cost. Variable cost per unit for each additional unit sold. C. and use of productive facilities. Cost-volume-profit relationships that are curvilinear may be analyzed linearly by considering only A. unit sales. the degree or amount that revenues may decline before losses are incurred is the A. The amount of decline in sales before losses occur is margin of safety. Mathematically. B. Marginal costing and CVP analysis Marginal rate of return.  Margin of safety is the excess of sales over the breakeven point.Chapter 4 B. To be relevant in marginal costing analysis. total costs should be separated as to either fixed or variable. VC÷ (SP – FC). Needed for determining product contribution. Margin safety . 95 Target (hurdle) rate of return. Total unit costs are A. Relevant for cost-volume-profit analysis. C ? A choice that describes total unit costs. B. margin of safety equals actual (or budgeted) sales less breakeven sales. 17. Irrelevant in marginal analysis.The rate or amount that sales may decline before losses are incurred is called: A.  If operations are in excess of breakeven point. Therefore. B ? Computation of breakeven point in units. C. 14. C ? The amount of decline in sales before losses occur. D. what is the breakeven sales level in units? SP = selling price per unit FC = total fixed costs VC = variable cost per unit A. (aicpa) D. Choiceletter “d” is incorrect. 15. it is the maximum amount of reduction in sales before losses are incurred. Choice-letter “b” is incorrect. (cma) D. C. Variable sales ratio. there is profit. what is important is the variable cost. FC ÷ [1 – (VC ÷ SP)]. C ? The rate or amount in sales before losses are incurred. Residual income rates. . (rpcpa) 15. Choice-letter “c” is correct. FC ÷ (SP – VC). C.  Choice-letter “c” is correct. Independent of the cost system used to generate them. Breakeven point 17. Given the following notations. And total unit cost is not needed in determining the contribution margin. (rpcpa) 16. D. Choice-letter “a” is incorrect. Sensitive level of income. Total unit costs are dependent of the cost system used to generate them. B. Total unit cost per se is not relevant in marginal costing analysis. total unit costs are irrelevant in marginal analysis. SP ÷ (FC ÷ VC). 16. 45. Decrease by less than the percentage increase in selling price. sales ratio is always equal to 100%. Questions 19 and 20 are based on the following selected budgeted data of Russel Gil Company for the coming year: Selling price per unit P 12. 35. breakeven point (BEP) in units and in pesos are computed as follows: BEP (units) = Fixed costs / Unit contribution margin or FC / (USP – UVC) BEP (pesos) = Fixed costs / Contribution margin ratio or FC ÷ (100% .500. A company’s breakeven point (BEP) in pesos of revenue may be affected by equal percentage increase in both selling price and variable cost per unit (assume all other factors are constant within the relevant range). Therefore.000 B. Variable cost ratio (VCR) is basically variable cost over sales.  The equal percentage increase in both sales price and unit variable cost does not change the contribution margin ratio. C. P12 – P8).  Breakeven sales in units equals total fixed costs over unit contribution margin.000 Variable cost per unit 8.00 (i.. assuming all other factors are constant. 30% . D ? Effect to breakeven point of equal percentage increase in both sales price and unit variable cost. Remain unchanged. the breakeven point will not change.000 Fixed expenses 150. breakeven sales in units is 37. 37. Choice-letter “d” is correct.VCR) In the marginal costing analysis.000 / P4 = 37. Unit contribution margin equals unit sales price (USP) less unit variable costs (UVC). (cia) 18.000 19.00 19. 15% C.Chapter 4 Marginal costing and CVP analysis 96  Choice-letter “b” is correct. 18.00 Budgeted sales 600. computed as follows: BEP (units) = P150.500 units 20. What is the margin of safety ratio in percent? A.500 C. B ? Breakeven sales in units. D. The equal percentage changes in selling price and variable cost per unit will cause the breakeven point in pesos to A.000 D. B. Therefore.e. Decrease by more than the percentage increase in the selling price. What is the breakeven in sales in units? A. 40. The unit contribution margin is P4. Increase by the percentage change in variable cost per unit. Therefore. as summarized below: Revenues (P12. 20% (rpcpa) Marginal costing and CVP analysis D. Therefore.50 per unit) P300. Considering the given data. 16.000).g. P180.000 / P600. Total fixed cost equals CM plus operating loss.  The company is operating at a loss.000). D ? Margin of safety ratio.000 C.Chapter 4 B.000 (e.. The additional units to sell to breakeven is the difference between breakeven point and the present actual units sold.000 units) 22.000) How many additional units should have been sold in order for the company to break even in 2004? A.000).e.Actual units sold (P300. 25% 97 20. 24.g.000 units Alternatively.000 / P12.e. we have: Budgeted sales P 600.000 Therefore. It has to increase its actual units sold to breakeven.. P40. P120.50 x 40%). 21. 8.000).000 Variable costs 180.000 Less: Breakeven sales (37. D ? The number of additional units to sell to breakeven.500 units x P12) 450.000 / P5 = 8. Therefore.000 Operating loss (40.000 units . The additional units to sell is calculated as follows: Breakeven sales (P160. MSR is 25% (i. P150.g.000 Needed increase in units sold to breakeven 8..000 / P5) 32.. and the UCM is P5 (e. A company is concerned about its operating performance.000 D. CM is P120.000 (cia) 21.. P300.g..50) 24. P12. the increase in units needed to breakeven is 8000 units determined by dividing the loss by the unit contribution margin (i.000 Margin of safety P 150.'s operating percentages were as follows: Revenues Cost of good sold Variable 50% Fixed 10 Gross profit Other operating expenses Variable 20 Fixed 15 100% 60 40% 35 . 32. the CMR is automatically 40%. The variable cost ratio is 60% (e. total fixed cost is P160.000 (e.  Margin of safety ratio (MSR) is margin of safety divided by actual (or budgeted) sales.000 + P40.000 B.000 – P180. Kent Co.000/P300. 900. At what revenue level would Kent break even? A. .667 D.000 (i. P 883. P10.750. 24.500.000 D. 53% B. The breakeven point in pesos is: BEP (pesos) = P500. what is the contribution margin percentage? A.0 million. Total fixed cost is P500.000 (P750.. B ? The contribution margin percentage. The total variable cost ratio is 70% (i.250. Direct materials.000. P1. P1. 50% + 20%).e. BEP in pesos is P2.000 B.  BEP in pesos equals fixed costs divided by CMR.000 x 25%). P1. P8.e.  BEP is fixed costs divided by CMR. B ? Breakeven sales.666.333 (aicpa) 22. The CMR is 30% (that is. Finally.000.000 (rpcpa) 23. total fixed costs and expenses equal P750.875. P3.000 C.000. 74% (rpcpa) 24. For the period just ended Chanda. P1. B ? The breakeven sales in peso. 100% sales ratio less 70% total variable costs and expenses ratio). If the sales value of 500 units is P102.. direct labor.500. P2. Inc. P1.000 B.000 / 30%). at what level is break-even sales? A. The fixed cost rate on sales totals 25% (fixed overhead of 10% plus fixed expenses of 15%).000 (P3 million x 25%).667 23. 44% C. 47% D. Therefore.000. CMR is 30%.850. Therefore.666.Chapter 4 Marginal costing and CVP analysis 98 Operating income 5% Kent’s sales totaled P2 million. A company produced 500 units of a product and incurred the following costs. generated the following operating results in percentages Sales 100% Cost of sales Variable 50% Fixed 10% 60% Gross profit 40% Operating expense Variable 20% Fixed 15% 35% Operating income 5% Total sales amounted to P3. P45..000 overhead (20% fixed).000 C. P2.000 / 30% = P1. 25. C.000 x 80%) ( 36.000 B.000. P750.000) Direct labor ( 10. B ? The amount of variable expenses at breakeven point. Selling price less unit fixed costs equals contribution margin D.e.000 The CMR is 47% (i. P250. B ? The formula used in determining the breakeven point (BEP). the total variable expenses at the break even point would be: A. C.000 D. P350. P450. contribution margin ratio at 25% and fixed cost at P250.Chapter 4 Marginal costing and CVP analysis 99  Contribution margin percentage (CMR) is the quotient of contribution margin and net sales.000/P102.000 x VCRatio (100%. BEP is determined as fixed cost divided by CMRatio. Which of the following will result in raising the breakeven point? A. At BEP.  Choice-letter “b” is correct.000).000 Or variable expenses (P250. (rpcpa) 26.000 / 25% x 75%) P 750. B..000. 27. Total fixed costs equal total revenues.000 Direct materials ( 8. Unit contribution margin times the break-even number of units equals fixed costs. Choice-letter “d” is definitely incorrect. An increase in the semi-variable cost per unit. Given the data on the problem. Which of the following formulas is used to determine the break-even point when using the contribution margin method? A.000/25%) P1. the contribution margin is determined as follows: Sales P102.  Based on the data given.000 (rpcpa) 25. Given the selling price at P120 per unit. A decrease in the variable cost per unit. Revenues less operating income equals variable costs plus fixed costs. total fixed costs equal to contribution margin where fixed costs equal unit contribution margin times the number of breakeven units. P48. the variable expenses at the BEP may be computed by multiplying breakeven sales and VCRatio.000 26. Choice-letter “a” is incorrect because it does not have relevance in the planning and control of costs and profit. BEP (P250. B. Choice-letter “c” is incorrect because sales less fixed costs is the sum of variable costs and profit (it also has no relevance).000) Contribution margin P 48. An increase in the contribution margin per unit.25%) 75% Variable expenses P 750.000) Variable overhead (P45. .000 C. and increase in BEP. Choice-letters “a” and “c” increase unit contribution margin resulting to decreased BEP. Increase in total and increase as a percentage of net sales. fixed costs must be decreased or UCM must be increased (choiceletter “c”: is correct). A calculation used in CVP analysis is the break-even point.Chapter 4 Marginal costing and CVP analysis 100 (cia) D. Choice letter “d” will not affect BEP. D. B ? The one that will raise breakeven point. 29. D ? Effect to semi-variable cost if net sales decrease by 15%. Contribution margin per unit for each additional unit sold.  BEP is fixed costs over UCM. To increase BEP. D. NTQ’s semi-variable costs would A. Increase in total. but decrease as a percentage of net sales. C. but not in direct relation to the change amount of sales (say. C. 28. Decrease in total. NTQ Inc’s net sales in 2006 were 15% below the 2005 level. (rpcpa) 28. (rpcpa) . Gross margin per unit for each additional unit sold. fixed cost should increase and unit contribution margin should decrease. B. Decrease both the fixed costs and contribution margin.  Breakeven point is fixed costs over unit contribution margin (UCM). if net sales decrease by 15%. To reduce the break-even point. Therefore.  Total semi-variable costs change. Once the breakeven point has been reached operating income will increase by the A. 27. Therefore. A semi-variable cost per unit is more of a variable cost than that of fixed costs. including choice of product-lines. Increase both the fixed costs and the contribution margin. Decrease the fixed costs and increase the contribution margin. Increase the fixed costs and decrease the contribution margin. Cost-volume-profit analysis is a key factor in many decisions. C ? The technique in reducing the breakeven point (BEP). marketing strategy. Sales price per unit for each additional unit sold. semi-variable costs may increase but not by 10%). if net sales increase by 10%. but increase as a percentage of net sales. Decrease in total and decrease as a percentage of net sales. B. and utilization of productive facilities. Fixed cost per unit for each additional unit sold. the company may A. an increase in semi-variable cost would decrease unit contribution margin. D. (rpcpa) 29. C. Choice-letter “b” is correct. the total semi-variable costs will also decrease but its percentage relationship to net sales will increase because the percentage decrease in net sales is greater than the percentage decrease in semivariable costs. B. pricing of products. 30. to reduce the BEP. A decrease in income tax rates. The installation of new computer-controlled machinery and subsequent layoff of assembly-line workers. the operating income will increase by every peso of increase in the amount of contribution margin. Petrol is the company’s most profitable product. Choice-letters “b” and “d” reduce unit contribution margin and increase BEP. 32. Choice-letter “d” is incorrect because unit gross margin does not reflect yet the net profit as variable selling and administrative expenses are still to be deducted. An increase in anticipated sales of petrol relative to sales of septine and tridol.  Choice-letter “b” is correct. septine and tridol. Fixed costs are assumed to be constant even beyond the BEP. An increase in anticipated sales of Simpol relative to the sales of Plutex and Coplex D. At BEP. Total contribution margin is equal to unit contribution margin (UCM) multiplied by the units sold.  Choice-letter “c” is correct because an increase in the anticipated sales of petrol relative to sales of septine and tridol will increase the weighted average unit contribution margin. Choice-letter “c” is incorrect because fixed cost per unit does not affect the overall profit of the business. Simpol. as contribution margin increases (due to the continuing increase in the number of units sold) with fixed costs as constant. any increase in the contribution margin is an increase in profit. Cook Company sells three chemicals. Simpol is the most profitable product while Coplex is the least compatible. C ? The event that will decrease BEP. total fixed cost is equal to total contribution margin. An increase in petrol’s raw material cost. Choice-letter “a” is incorrect because sales price is only an increase in revenue and not yet in profit. Choice-letter “a” would increase fixed cost and breakeven point. A. once the BEP is reached. An increase in Simpol raw materials (rpcpa) 32. (cma) 31. after the BEP. An decrease in Coplex’s selling price C. though it has great impact in the establishment of a competitive unit sales price. D. Plutex and Coplex. which will decrease BEP. B ? The increase in operating income once the breakeven point (BEP) is reached. 31. B. An increase in the overall market of Plutex B.Chapter 4 Marginal costing and CVP analysis 101 30. C ? The event that will decrease the overall breakeven point (BEP). Which one of the following events will definitely decrease the firm’s overall breakeven point for the upcoming accounting period? A. Marston Enterprises sells three chemicals: petrol. Therefore. tridol is the least profitable. Which of the following events will definitely decrease the firm’s overall BEP for the upcoming account period. . A decrease in tridol’s selling price. C. C ? The number of units to breakeven. Coplex is the least profitable..800 units. Choice-letter “b” is incorrect because a decrease in Coplex’s selling price would aggravate a decrease in the average UCM and will increase the composite BEP. Simpol is the most profitable. would hardly change the average UCM and that of breakeven point.000 Cashflow break-even 600.331. P1.000 Break-even sales 2.e. and Phines is at the middle.000 (rpcpa) C..720 General and administrative P3. P5.100. Total fixed costs is P9. The total unit variable cost is P24 (i. P2.000 Budgeted contribution margin 1.e.000 D. B.Chapter 4 Marginal costing and CVP analysis 102  Composite (or overall) BEP is equal to fixed costs divided by the average UCM.200 units.000 The company’s margin of safety is A.598. 388. Choice-letter “d” is also incorrect because an increase in Simpol’s raw materials cost would mean a decrease in UCM and an increase in the composite BEP.480).. Therefore.720 + P3.400 units. P22 + P2).732. Estimated cost data regarding this product and other information for the product and the companies are as follows: Sales price per unit P40 Total variable production cost per unit P22 Sales commission (on sales) 5% Fixed costs and expenses: Manufacturing overhead P5.200 (i.000 B.200 / P16 = 583.000.732.200 units 34. (cia) 33. the BEP in units is: BEP (units) = P9.800. and the unit contribution margin is P16 (i. the fixed costs must decrease or the average UCM must increase.500. To decrease the composite BEP. A company manufactures a single product.480 Effective income tax rate 40% The number of units the company must sell in the coming year in order to reach its breakeven point is A. Fely Company reported the following for the year just ended: Budgeted sales P 3. The company produces and sells three products. and thereby would decrease composite BEP. the product at the middle of the sales mix profitability. P40 – P24).331.200.000 units. D. 518. 972.000 . P1. P 900. 33.598.400. Choice-letter “a” is incorrect because an increase in overall market for Phines. Choice-letter “c” is correct because an increase in the anticipated sales of Simpol relative to the sales of Plutex and Coplex would trigger an increase in the average UCM. 583.e.  The sales commission is P2 per unit (5% x P40). C. The following data refers to cost volume profit relationships of Trilogy Co: . C ? The contribution to profit by the 1.000 How much will be contributed to profit before income taxes by the 1. as follows: Budgeted sales P3.001st unit sold? A. C.000 .000 / 1.001st unit sold. A ? The amount of margin of safety. or. (aicpa) 35. Breakeven point decreases.  CMR increases when unit sales price increases or unit variable cost decreases. Breakeven point increases. B. the change in BEP is not automatically due to the change in the CMR. Variable costs as a percentage of net sales increase. And unit contribution margin is total fixed costs divided by breakeven units. 36.  Margin of safety is the difference between actual of budgeted sales and breakeven sales.000.000 35.000 units = P150 37.000 P 500 P150. The profit contributed by the first unit after the BEP is the unit contribution margin. The unit contribution margin is determined as follows: UCM = Fixed costs / BEP units UCM = P150. Choice-letters “a” and “b” are incorrect. P 0 (aicpa) 36. contribution margin equals fixed costs. in effect. profit increases by every peso of increase in contribution margin. The contribution margin ratio always increases when the A. automatically CMR increases.001st unit sold is the very first unit after the breakeven point. P150 B. C ? The event that makes the contribution margin ratio (CMR) to increase. At breakeven point.  The 1. Choice-letter “c” is correct. The following information pertains relationships: Breakeven point in units sold Variable costs per unit Total fixed costs to Nova Company’s cost-volume-profit 1. P650 C.100. P500 D. when unit contribution margin increases.Breakeven sales 2. as variable cost ratio decreases.Chapter 4 Marginal costing and CVP analysis 103 34. Besides.000 Margin of safety P 900. D. CMR affects breakeven point and not the BEP affecting the CMR. Beyond the breakeven point. Variable costs as a percentage of net sales decrease. 000. Net sales price is 160% of variable cost less 10% markdown. A ? The one that decreases contribution margin the most. 37.000 Variable costs per unit P 250 Total fixed costs P75.6% D. P325 D.5% C. (cma) 39. the CM is equal to fixed cost. each unit sold increases profit by the amount of the unit contribution margin (UCM). If the markdown average 10%.7% (cia) 38. A 15% decrease in selling price. P250 C. A 15% increase in variable costs. an increase in variable cost or a decrease in unit sales price and unit variable cost by the same amount. C.6% 39. The variable cost ratio (VCR) and contribution margin ratio (CMR) are calculated as follows: VCR = 100% / (160% x 90%) = 69.  Contribution margin is the difference between sales and variable cost. And. Which of the following would decrease unit contribution margin the most? A.001st unit sold. B.5% B. Choice-letter “a” is the correct choice.000 units = P75 38.000 How much will be contributed to operating income by the 1. The UCM is P75 and this is also the amount of profit on the 1. A retail company determines its selling price by marking up variable costs by 60%.4%. Inasmuch as the sales price is assumed to be greater than the variable cost. A 15% decrease in fixed costs. D.001st unit sold is the first unit sold after the breakeven point. A 15% decrease in variable costs. Contribution margin is decreased by a decrease in sales price. UCM = P75.44% = 30.69. the company uses frequent selling price markdown to stimulate sales. . which is the 100%. In addition.001st unit sold. The UCM is equal to contribution margin over the number units sold. a 15% decrease in sales price (choice-letter “a”) would decrease contribution margin more than the decrease effected by a 15% increase in variable cost (choice-letter “b”). B ? The contribution margin ratio. Zero (rpcpa) 37. At the BEP.  The 1.Chapter 4 Marginal costing and CVP analysis 104 Breakeven point in units 1. After the breakeven point. C ? The contribution to operating income by the 1. Choice-letters “a” and “b” would decrease contribution margin. P75 B. 27. CMR = 100% . what is the company contribution margin ratio? A.000 / 1. 30.  The sales price is based on variable cost. 41. Variable cost ratio is variable cost over net sales price.001st unit sold? A. The fixed cost is given at P75. 72% (rpcpa) 41.Breakeven sales Margin of safety Amount P82. 3.e. Questions 40 and 41 are based on the following information.875 The margin of safety ratio is 20% (i.500) 3. the amount by which sale can decline before losses occur is known as the A.500 Breakeven point in peso sales P65.  Margin of safety ratio is margin of safety over sales.e. 20% C. Actual sales (19.343. What was the margin of safety percentage for the last quarter of Lan? (rounded to the nearest percent) A.25) . then the margin of safety ratio. P 0.00 P16. the MSR may be determined using the data in units and still arrive at 20% (i.75 65.20 41. Sales volume variance.75 Units 19.25 . P16. P 1.375 15.375 Breakeven point in number of toys 15.  Unit variable cost is the difference between unit sales price and unit contribution margin. 28% B. Lan Pala Tropical Stuff Toys manufactures and sells dolls. Variable sales ratio.500 3.875 Total fixed costs P47. A ? The margin of safety ratio (MSR).375 x P4. At breakeven point..875 / 15. For a profitable company.875/19. while choice-letter “d” does not affect contribution margin.468. What was Lan’s variable cost per doll? A.468.275 40.20 D.75). P 3..96 40. C. fixed costs equal the contribution margin.375).Unit contribution margin (P47.500) P4. 25% D. The following information relates to the operating results for the last quarter: Stuff toys sold 19. Therefore.75 / P82. let us get the margin of safety.Chapter 4 Marginal costing and CVP analysis 105 Choice-letter “c” would increase contribution margin.05 Unit variable cost P1. C ? The amount of variable cost per doll.343. First.05 (rpcpa) C. 42. P 4.875. the unit variable cost is computed as follows: Unit sales price (P65. .275 / 15.25 B. Alternately. (rpcpa) D.  Breakeven point is fixed costs divided by unit contribution margin.250 B.50) P1.600 D. there is still no loss. Choice-letter “c” is incorrect because sales volume variance refers to the net effect on profit of the difference in actual sales and budgeted sales. • Fixed cost will increase by 10%. Profit = Margin of Safety x CMR).000 (P1.050.00 Total fixed costs (20. and the variable costs to manufacture them were P22. This is called the margin of safety (choice-letter “b” is correct). Marginal costing and CVP analysis Margin of safety. and pretax profit of P150.33) Unit contribution margin P 52.250 (rpcpa) 43.155. At BEP. 2. 19. • The income tax rate will remain unchanged. the amount of decline in sales before losses occur is the difference between the actual (or budgeted) sales and the breakeven sales.50 per unit. • Variable manufacturing costs per unit will increase by one-third. the data are treated as follows: Before After Unit sales price P 75. Choice-letter “d” is incorrect because marginal income rate is a loose description that relates to the relationship of margin over sales and not on the amount of a reduction in sales. variable costs of P300. For the company to break-even the coming year. 21. The net income last year was P50. B ? The amount of decrease in sales before losses occur.00 Unit variable costs 22.00 (P22.250 units (P1. Marginal income rate.600 C. Using the “beforeafter analysis”. The company needed to sell 20. Therefore.050. margin of safety is the amount of sales in excess of BEP.000 key rings to break-even. Last year. and eventually. 106 42.g. Analyzing it in a different perspective. 000.00 P 90.50 30. Inc.400. the company should sell A.000 / P60).  Losses occur when sales fall below the breakeven point (BEP). 000. reduced . profit (e.50 P 60.000 x P52. 43. manufactures and sells key rings embossed with college names and slogans. Choice-letter “a” is incorrect because variable sales ratio (or variable ratio) is the relationship of variable costs over sales. A company has revenues of P500. the key rings sold for P75 each.000 x 110%) The new breakeven point in the coming year shall be 19.50 x 1. It suggests that this excess sales carries a corresponding contribution margin.Chapter 4 B. Bulacan Gold. 000.000 P1. 21. 44..155. D ? The breakeven point in the coming year. If the company increased the sales price per unit by 10%. The company expects the following for the coming year: • The selling price of the key rings will be P90. 00 Labor 1.000 (P50.55 (rpcpa) 46.000 (cia) 44. P 0. what would be the new breakeven point in pesos? A. the before-after analysis would be used as follows: Before After Sales P500. a manufacturing company. P 0.45% (P250.000 (same) Contribution margin P200.50 What is the expected contribution margin per unit on the new order? A. and left variable cost per unit unchanged. If the order will be accepted.Income before income tax 320. P200.000 46. P230. an offer for a new order at P8. A ? The new breakeven point in pesos. P520.40 C.26) P 520.000 (P500.000 / P550.000 (rpcpa) 45.000 C. P 0.000 300.Chapter 4 Marginal costing and CVP analysis 107 fixed costs by 20%. A ? The amount of fixed costs.000 x 110%) -Variable cost 300.000 (P40.000 P 40.000 P550. B .777 B.75 Variable overhead 1.000 B.000 with a sales volume of P2 million.000 D.80 per unit.60 D. It aims to have a net income of P320. The variable cost per unit of production of Asian Corporation follows: Materials P 4.  Inasmuch as there are changes in the variables of profit. P 0. P110.  Fixed costs and expenses may be determined by getting the difference of contribution margin and income before income tax. is operating at 90% capacity.75 Total P 7. Since there is no other use of the 10% idle capacity. P100.000) Fixed costs P 50.50 B. the 5% additional capacity will be sub-contracted at the cost of P7.200 D.000 / 45.000 C.000 x 80%) New BEP (pesos) P 88.45%) 45. P125. P 88.20 per unit requiring 15% capacity is being considered.000 CMR 45.000 P250. P 83. Mela Corporation has a contribution margin ratio of 0.26.000 Total fixed costs and expenses P 200. Its total fixed costs amount to A.000 . Asian Corporation. as follows: Contribution margin (P2 million x . The new contribution margin is calculated as follows: Sales last year (P550. as follows: 10% capacity 5% capacity (Regular) (Sub-contract) Unit contribution margin P8. 30% D. How will these changes affect the breakeven point? A. C. B. 000.  The new contribution margin ratio is determined by dividing the new contribution margin with the new sales.13 Average unit contribution margin P0. The average unit contribution margin may now be determined as: Regular sales (P0.000 and variable cost per unit remained unchanged. 20% C. The effect cannot be determined from the information given. Last year.20 P8. A company increased the selling price of its product from P1.40 x 5/15) 0. One for the 10% capacity and the other for the 5% capacity. a 10% increase over last year. fixed costs are expected to be P120.000 Add: Increase in contribution margin 15.000 CM this year 165.Chapter 4 Marginal costing and CVP analysis 108 ? The expected unit contribution margin on the new order. respectively.000.60 47. 000.  There are two unit contribution margin here. the contribution margin ratio must be A.40 The individual UCM shall be multiplied by the production mix of 10% and 5% for regular production and sub-contract.000/110%) P500.47 Sub-contract (P0.20 .70 x 10/15) P0. B ? The contribution margin ratio to increase operating income by P15.Unit variable costs 7.70 P0.000 Divide by sales this year 550. The breakeven point in units will be increased.80 Unit contribution margin P0. D. This year. (aicpa) 48. and revenues are forecasted at P550.000 to P480. 40% B. .000 in the coming year. The breakeven point in units will remain unchanged. 70% (aicpa) 47.00 to P1. the contribution margin ratio of Lamesa Company was 30%. the same as last year.000 x CMR last year 30% CM last year 150. For the company to increase operating income by P15.10 a unit when total fixed costs increased from P400.50 7. D ? Effects to BEP of a change in unit sales price (USP) and fixed costs (FC).000 CMR this year 30% 48. The breakeven point in units will be decreased. a unit variable cost of P 0.e.10 0. . let us use the following data: a.. Since Company 2 has a higher profit (e. Sales with profit 50.70 0.  Company 1 has a higher CMR and lower fixed costs which yield to lower BEP compared that of Company After the BEP. a unit variable cost of P0. making its fixed costs 40% higher than those of Company 1. Lower Lesser C. The effect cannot be determined from the information given because the amount of unit variable cost also affects the change in the BEP. C.10 Unit variable cost 0.000 685.333 1. 30%). List A List B List A List B A.667 New Data P1.70 Unit contribution margin P0.40: Original Data Unit sales price P1. Predicted operating loss. Company 2 also has a contribution margin ratio that is 30% greater than that of Company 1. 40%) than that of Company 1 (e. Two companies produce and sell the same product in a competitive industry.. To illustrate.60 Fixed costs P 400.) Say. Company 1 will have the <List A> breakeven point in terms of pesos sales volume and will have the <List B> peso profit potential once the indifference point in peso sales volume is exceeded.000 (BEP decreases) b. while an increase in FC increases BEP.30 P0. The unit variable cost remains constant and there is no exact variable cost data given to precisely determine the quantitative effects of the given change in USP and FC to BEP.000 Breakeven units 1.g. their BEP and effects to operating profit beyond the BEP.40 Unit contribution margin P0.000 P 480.Chapter 4 Marginal costing and CVP analysis 109  An increase in USP increases unit contribution margin (UCM) and decreases BEP. Profit = Margin of safety x CMR). which of the following should be deducted to fixed cost in the numerator? A. A ? Comparing Company 1 to Company 2. the company having higher CMR is expected to register higher profit (i. Company 2 is more automated.00 Unit variable cost 0.70: Original Data New Data Unit sales price P1.00 P1. Thus. Unit contribution margin.000 Breakeven units 666. By comparison.200. Higher Greater (cia) 49.333. Lower Greater D.40 Fixed costs P 400. Company 1 has a contribution margin ratio of 40% and fixed costs of P25 million.715 (BEP increases) 49..70 P 480.) Say. In using cost-volume-profit analysis to calculate expected unit sales.40 P0. Higher Lesser B. its increase in profit will tend to be greater after the BEP. the selling price of the product of each company is the same.g. 000 (i. (aicpa) D.000 units or more. The target sales if after tax income is P123. 833. P200 – 60).000. A ? The one added to fixed costs in the numerator in CVP analysis.334 D. the targeted sales in units is 11.000.e. 785. 15.000. Therefore..500. Choice-letter “d” is incorrect because variable cost is not related to fixed cost. The income tax rate is 30%. That is. B. Choice-letter “b” is incorrect because operating income is added from fixed costs in the numerator in calculating sales with profit. Marginal costing and CVP analysis Predicted operating income.000.334 units 52.750 units.000.000) / P3 = 833.000.00 (P15 x 20%).000 + P420. sells Product O to retailers for P200. And IBIT is P176. P40 + (10% x P200)]. And the UCM is P140 (i.000.5 million at 500..420. The contribution margin percentage is 20%. The total of fixed costs and expenses is P1.11. (rpcpa) 52.500. 13.e.500.000 units of output and P1.  The total unit variable cost is 60 [i.000. D ? The target sales if the after tax income is P123.500.200 would be A. Ipil-ipil Company would like to market a new product at a selling price of P15 per unit. Inc.000.200 / 70%).000 + P1.400 units computed as follows: Sales = (FC + IBIT) / UCM . the company must product and sell more than 500. The UCM is P3.000 units.000.400 units.000 (i. to generate an earnings of P1.530 B.000).640 units. C. Variable costs. Therefore.000 is 833. 754. 110 50. Fixed manufacturing costs total P1. P1. B ? The number of units to be sold to earn an income of P1 million.  Choice-letter “a is correct.e.000 units x P3) less fixed costs of P1. P123. Choice-letter “c” is incorrect because unit contribution margin serves as the denominator in the computation of sales in units. The unit variable cost is P40 with a selling commission of 10%.000 units of production and P1.000 for less than 500.000.000 for 500. Fixed costs for his product are P1.. the operating income shall be P500.000 (500. D.320 (rpcpa) 51.000 per month while fixed selling and administrative costs total P420. the predicted operating loss is deducted from the fixed costs in the numerator in computing sales with profit.. If the company produces and sells 500. 51.000.950 units. The sales in units to product a profit of P1. Merchandiser. 10.Chapter 4 B. P1 million at less than 500. How many units of this product must be sold to earn a target operating income of P1 million? A. 825. CM of P1.  There are two fixed costs given.900 C.200.e.000 or more units of output.334 units computed as follows: Sales = FC + IBIT / UCM = (P1.000 units and the fixed cost to be incurred shall be P1. the sales volume necessary to breakeven is P750. 35% C. This means that at these sales there is no profit or loss and the change in profit is also zero. The CMR of 35% is determined as follows: CMR = Δ FC / Δ Sales (i.00 with variable costs of P300.000. A ? The profit-volume ratio.000 but with the expected increase in final sales.e. 54.  The profit-volume ratio is the same as the contribution margin ratio.000 and an operating loss of P20.000 (i. .000. Questions 54 and 55 are based on the following information.000. it is anticipating that the selling price per unit and the variable expenses will not change.000 (rpcpa) 55.10%) = P800.750. P2.. What is the profit-volume ratio of Laguna Marketing? A.750 upon moving their place of business to the downtown area. P400. What would be the total fixed costs of Laguna Marketing after the increase of P78. Note that sales given in the question are at breakeven point. fixed costs of P120.000) = 35% 55.000. At the present. the computation of the sales with profit shall be: Sales = Fixed costs / (CMR – NPR) = P120.000 C. Nette & Company has sales of P400. And CMR is contribution margin divided by sales.420. any change in fixed cost is also the change in contribution margin. the sales are at BEP) = P78.183. the CMR is 25%.000-P750.000 – P400. Likewise.250 C.e.750 B.000 D. P500.000).000 + P176. therefore.e. P300.Chapter 4 Marginal costing and CVP analysis 111 Sales = (P1.000/P400.750 / (P975. P 300.000 (rpcpa) 53. the VCR is 75% (i.. and.. P262. Laguna Marketing Company is expected an increase of fixed costs by P78.500 D.400 units 53. By how much would Nette need to increase its sales in order to achieve a target operating income of 10% of sales? A. 45% B. P800.000 / (25% . P462. Based on these projections. Since the profit ratio is given.  The present sales amount to P400.000) / P140 = 11. the sales volume necessary to breakeven would go up to P975. P800.000). 40% D. Therefore. A ? The fixed cost after the increase of P78.000 The increase in sales shall therefore be P400.750? A.000 B. A ? The amount of increase in sales to achieve a target profit of 10% of sales. 65% (rpcpa) 54. P341. e.Chapter 4 Marginal costing and CVP analysis 112  The given sales are at breakeven point.000 x 35%). A ? The maximum amount of advertising expense to obtain a profit of P200. how much will fixed costs increase to have a gross margin equal to 10% of the sales value of the additional cost of 50.000 Increase in fixed cost P50. The unit sales price is P5 (i. P800.500. P 125.000 units at the same price and contribution margin.000 units x (10% x P5)] 25. P 57. The new fixed cost is not readily given and must be computed.000 (i.000). operating income) is 10% of sales..g.000 .000. Contribution margin is 30%. The gross margin (e.  Increase in fixed cost is new fixed cost less old fixed cost. P 50.000 Selling and administration P 700.500 B.000 units to be sold? A.000 An advertising agency claims that an aggressive advertising campaign would enable Birney to increase its unit sales by 20%.. What is the maximum amount that Birney can pay for advertising and obtain an operating profit of P200.. P975..  Increase in contribution margin less increase in operating income is increase in fixed cost.500 C.000 D. fixed cost equals contribution margin. P100.000.e.000 B.000 (aicpa) 57. The fixed costs P341.50.000 units Sales 150.e. Breakeven sales is P1 million. Birney Company is planning its advertising campaign for 2006 and has prepared the following budget data based on a zero advertising expenditures: Normal plant capacity 200. B ? The amount of increase in fixed cost.000 units Selling price P25 per unit Variable manufacturing costs P15 per unit Fixed costs: Manufacturing P 800.000 + P700.50).000 (rpcpa) 56. P3. The increase in fixed cost is P50. P300.000 C. calculated as follows: Increase in CM (50.50 / 70%) and the UCM is P1. P 67.50 (P5 – P3.Increase in operating income [50. P15 – P10).e.000 D. To sell an additional 50. Variable cost per unit is P3. P200. P550.250 (i.000.. A.50) P75. The fixed costs after its increase shall be the contribution margin in the second year with breakeven sales of P975. 56.000 57. At BEP. .000 units x P1. And sales are expected to increase by 20%. The UCM is P10 (i.000. The old fixed cost is P1. C. the breakeven point in units (rounded to the nearest hundred) for surge protectors is A.300 units.  Sales in units are fixed costs plus operating profit divided by unit contribution margin.00 + P0. 20.00 Distribution .000 + P30.25 Direct labor 4.000 units.000 units.700 units. C.000 units (i. 15.000 units.e.600 units.000 units x 120% x P15) . D..000 additional operating income before taxes? A.800.000 200.000 1.000. D ? Breakeven point in units.75 The company will also be absorbing P120. B. surge protectors for high-voltage electric flows. Unit Costs Direct materials P 3.000 units.600. Therefore. 8. is developing a new product. Therefore. (cma) 59.000) / P6 = 25.000 of additional fixed costs associated with this new product.Old fixed costs Increase in fixed costs P1.000 units . The cost information below relates to the product. B.000 currently absorbed by other products will be allocated to this new product.75). D. Allocated fixed costs are not included in the determination of the product’s breakeven point. Allocated fixed costs. sales is unit is 25.e. How many surge protectors (rounded to the nearest hundred) must Bruell Electronics sell at a selling price of P14 per unit to gain P30.000/P8). A corporate fixed charge of P20.New operating income New fixed costs . 10.  The unit variable cost is P8 (i.000 units. 20. 28. and the unit contribution margin is P6 (i. C ? Total sales in units if operating profit is P30. 25. If the selling price is P14 per unit. P14 – P8). shall be included in the total fixed costs in the computation of overall company’s breakeven point. Bruell Electronics Co. 10. 59.. P120.000 Questions 58 through 60 are based on the following information. the BEP in units is 20. 58.25 + P4.000 1. however.500.000 P 100.000 units.Chapter 4 Marginal costing and CVP analysis 113 The increase in fixed cost shall be computed as follows: New contribution margin (50.. computed as follows: Sales = (FC + IBIT) / UCM Sales = (P120. (cma) 58.e. P3. C ? Sales in units if the after-tax income is P172. The organization is subject to an income tax rate of 30%. B.e. and the new unit variable cost is P24 (i.334 units. D ? Sales in units if the after-tax income is P30. 10.Chapter 4 Marginal costing and CVP analysis 114 60.  If the after-tax income is P30.000 ...000 (i. P504.000.50 with a higher-grade unit that costs P5.000) / P6 = 28.800 / 60%)..800.500 units Questions 62 and 63 are based on the following information. (cma) 61.000). The sales to achieve an aftertax income of P172. P172. which is subject to a 40% income tax rate.000 (i. as follows: Sales in units = (P120.000? Bruell Electronics’ effective income tax rate is 40%.e. P180. 800 in the upcoming period. B.. 21.300 units.e.  The income before income tax is P288.000 + P50. the income before income tax is P50. Austin will depreciate the machine over a 10-year life with no estimated salvage value by the straight-line method of depreciation.000 Rent 60. 28.000 units. The salespeople receive a salary plus a commission of 5% of sales.000 Salaries 180. C.300 units. D. Last year the organization’s net income (after taxes) was P100.50 and (2) acquiring a P180.000 (i. The sales price remains the same. An organization sells a single product for P40 per unit that it purchased for P20. Total fixed cost is increased by P18. (cma) 60. 22.800 is 22.000) / P36 = 22. D.700 units.500 units. 19.000 + P18. P30.800.000 Management plans to improve the quality of its sole product by (1) replacing a component that costs P3. the new unit contribution margin is P36 (P60 – P24).50 – P3.000 + P288.000 (i.000 / 60%). How many surge protectors (rounded to the nearest hundred) must Bruell Electronics sell at a selling price of P14 per unit to increase after-tax income by P30. 27.316 units. Therefore.500 units.000/10 years).000.e. Hence. 25. 20. P5..334 units 61. There is an increase in unit variable cost of P2 (i. C.e.50). the new fixed cost is P522. The fixed costs of the organization are Advertising P 124. calculated as follows: Sales (units) = (P522. A.000 units. had the following operating data for the period just ended: Selling price per unit P 60 Variable cost per unit 22 Fixed costs 504. 000 packing machine.. Austin Manufacturing.e. it must sell A. P22 + P2).000 units. therefore. If the company wants to earn after tax income of P172. the sales in units is 28. P40 – P22).000).e. P396.e.000 units.000 D. With a 10% increase in volume the net income after tax shall increase by P208. C ? Amount of sales to earn the same net income last year.e.000 B.800 units.000 – P80.000 x 10% x P16) P320. 22.000 (i.000 C. The fixed cost is reduced to P316. D ? Breakeven point in units.000 units.  Sales in pesos is FC plus profit divided by CMR. Video King Company sells video tapes.000 P396.  The unit variable cost is P22 composed of unit purchase cost of P20 and commission of P2 (i.e. 18.000 B.000 based on a sales volume of 200. P 40. The organization is considering changing the compensation plan for sales personnel. 8. P40 .630.000. P 48.000 64. The unit contribution margin is P18 (I.  First. P1.. And the BEP in units is 30. The variable cost consists of P40 unit purchase price (bulk orders) and a handling cost of P8 per unit.000 units. P100.. P1.000 + P144.000 D.000 . B.e. D..150. 19. C. The breakeven point in unit sales for the organization is A. The projected after tax net income for the year is P480.Chapter 4 Other fixed costs Total Marginal costing and CVP analysis 32. The new unit variable cost is P24 [i.. (cia) 62.. P208. If the organization increases the commission to 10% of revenues and reduces salaries by P80.000 (cia) 63. what revenues must the organization have to earn to have the same net income as last year? A.000 units (i.e.000 (i.P24) or a CMR of 40% ([16 / P40). P1. P64 – P48). let us get the incremental contribution margin. An increase of 10 percent in projected unit sales volume for the year would result in an increased after tax income for the year of A. C ? The amount of increase in net income given a 10% increase in unit sales.e.400.000) / 40% = P1.350.. P20 + (10% x P40)]. then deduct the corresponding tax thereof. resulting to a new unit contribution margin of P16 (i.000 ÷ P18). The sales in pesos shall be determined as: Sales (pesos) = (FC+ IBIT) / CMR = (P316. P1. P396.800 / 70%).000 C. It sells the tapes at P64 each.800 units.000 115 62. P120. 5% x P40). 63..042.000 computed as follows: Incremental CM (200. Annual fixed cost are P2.150.000 (rpcpa) 64..e. The income before income tax (IBIT) is the same as last year at P144.000 and the company’s income tax rate is 35%. The unit contribution margin is P16 (i. the composite breakeven point in pesos may also be determined by getting the sum of individual product sales.000 B.000 units 66.000 D.000 units.000 C. and average CMR is average UCM divided average USP. P300.. The data below pertain to two types of products manufactured by Korn Corporation: Unit sales price Unit variable costs Product Y P 120 P 70 Product Z 500 200 Fixed costs total P300.000 annually. C ? Composite BEP in units..  CBEP in pesos is fixed costs over average CMR. How much is Korn’s breakeven sales in units? A. 2. 857 C. The expected mix in units is 60% for product Y and 40% for product Z.000 B. the allocated BEP in units shall be 1. D ? Composite BEP in pesos. The average UCM is determined as follows: Sales Average UCM Mix Ratio UCM Product Y P 50 60% P 30 Product Z 300 40% 120 Total P150 And the CBEP is 2. P544.000 P208.000 116 . determined as follows: Product Y (P120 x 60%) P 72 Product Z (P500 x 40%) 200 Average USP P272 Average CMR is average UCM divided by average USP and is computed at 55.000 units computed as follows: CBEP (units) = FC / Ave. How much is Korn’s breakeven sales in pesos? A. P150 / P272).e.200 units for product Y (i.459 rpcpa) (aicpa / 65.Incremental tax (35%) Incremental profit after tax Multi-product sales Questions 65 and 66 are based on the following information.e. Average USP is P272. P420. 65.15% (i.000 / 55.. First. 1.  Composite breakeven point (CBEP) in units is total fixed costs over average UCM. P475.15% = P544.000 (aicpa) 66.000 . 2. given that the CBEP in units is 2.111 D. 2.000 Alternatively.000 / P150 = 2.Chapter 4 Marginal costing and CVP analysis 112. The CBEP in pesos shall be: CBEP (pesos) = FC / Average CMR = P300. UCM -= P300. Chapter 4 Marginal costing and CVP analysis 117 units x 60%) and 800 units for product Z (i. let us determine the composite breakeven point by dividing the fixed costs by the average unit contribution margin (UCM).000 units (i. The average unit contribution margin is calculated by multiplying the individual UCM of the products with their respective sales mix ratio. compute the CBEP in units. B ? The number of units of X to sell at the composite BEP. then..000 67. the sales mix is: Red = 3 .e. P600. but is to be obtained as follows: If: 3 Reds = 1 Blue Therefore: 2 Whites = 1 Red 6 Whites = 1 Blue And.00 per unit of White 2. P1.000 (aicpa) 67. How many units of X would Tomas sell at the breakeven point? A.e. Contribution margin are: P 1.00 per unit of Z.000 C. White. 400. 360.000 B. Maribel is selling three products: Red.50 per unit of Y.30 Y (3 x 2) 6 1.000 units (i.. Fixed costs are P720. 108.000. 2.50 6/10 0. and Blue.000 D. and two units of Y for each unit of X. Fixed costs are P600. D ? The number of units of White to sell at breakeven point. and Z. Tomas Company sells products X.  First.30 per unit of Blue How many units of White would the company sell at breakeven point? A.200 units x P120) P144.90 Z 1 3. 216.000 units x 40%). CBEP is fixed costs over average UCM.e. and two units of White for every unit of Red.50) and the share of product X is 120. 68.000 Product Z ( 800 units x P 500) 400.000 (rpcpa) 68. The CBEP in pesos shall be: Product Y (1.000 D.000 C. 360.000 / P1..30 10 P1. UCM X 3 P1.00 3/10 P0. and P3.00 per unit of X.50 Therefore. The contribution margins are P1. 72.  First. the sales mix ratio is not readily given. Y.000.00 1/10 0. the CBEP in units shall be 400. 120.000 units x 3/10). 40. Tomas sells three units of X for each unit of Z. allocate it among the products. Now.000 Composite BEP P544. 400. The company sells three units of Red for every unit of Blue.90 per unit of Red 2.000 B. The average UCM is determined as follows: Sales mix UCM Sales mix ratio Ave. 90 x 3/10 = P .e. 36.000 B.000 (rpcpa ) 69.23 Average UCM P2.e. Laboratorio Unico. .8 million/P45). The fixed costs are given.. P1. the share of each product on the CBEP shall be calculated based on sales mix ratio.Chapter 4 White = Blue = Marginal costing and CVP analysis 118 6 1 10 The average UCM is determined as: Red P1.20 Blue 2. the volume of C3 to be sold would be A.00). 24.00 x 6/10 = 1. 360. The company makes the following contribution margin per measure: C1 P30 C2 P45 C3 P90 Fixed costs amounted to P1. Inc.000 x 1/10). 40. The average UCM is determined by multiplying the individual UCM with their sales mix ratio.000 C.000 / P2.00 The composite breakeven point (CBEP) is 360. 12. P720. D ? The volume of C3 to be sold at breakeven point. as follows: 3 C1 = 1 C3 2 C2 = 1 C1 Therefore: 6 C2 = 1 C3 The sales mix ratio is: C1 3 = 3/10 C2 6 = 6/10 C3 1 = 1/10 10 The average UCM is C1 = P30 x 3/10 = P 9 C2 = 45 x 6/10 = 27 C3 = 90 x 1/10 = 9 Average UCM P45 The composite BEP is 40. 4.000 D. At break-even point. The composite BEP should be distributed among the products by their sales mix ratio.000 units (i.  Composite BEP is fixed cost divide by average UCM.000 units (i.8 million.000 units (i. The share of product White in the composite breakeven point is 216.e..000 units (i.e. It sells to industrial users who use and buy these chemicals in the following ratio: three (3) measures of C1 per one (1) measure of C3. two (2) measures of C2 per one (1) measure of C1. C3’s share in the composite BEP is 4.. 69. formulates and sells three major chemicals: C1.000 units x 6/10). With the CBEP already determined..57 White 2.30 x 1/10 = 0. Therefore. C2 and C3. Margarita Manufacturing Company produces two products for which the following data have been tabulated. 4.50 Fixed manufacturing cost P .500 units Questions 72 through 74 are based on the following information. 9. Fixed costs are P49.  Using the average UCM in question 70 as P1. 23. Considering the company as a whole. the unit sales must have been Product X Product Y Product X Product Y A.000) / P1.00 P1. A ? Composite BEP in units.71429 Product Y (P1. The average UCM is determined as follows: Product X (P2.800 59.57143.000 32.85714 Average UCM P1.20. respectively.00 Variable manufacturing costs P2.500 / P1.500 C.57143 = 45.  Composite BEP in units is fixed costs divided by average UCM. The unit contribution margins for X and Y are P2. 5.00 per machine hour. 8.00 .000.250 B.. 31.000 units Product Y (45.500 units 71. The sales mix consists of a composite unit of two units of X for every five units of Y (2:5).500 units The CBEP shall be allocated based on their sales mix as follows: Product X (45. If the company had an operating income of P22.57143 Composite BEP (pesos) = FC / Average UCM = P49. 70. A company sells two products X and Y. the number of composite units to break even is A.75 P 20 Variable selling costs P1.000. 28.500 D.57143 = 31. Per Unit XY-7 BD-4 Selling price P4. 13.600 71.500.Chapter 4 Marginal costing and CVP analysis 119 Questions 70 and 71 are based on the based on the following information.500 units x 2/7) = 13.00 P1. B ? Units sold if operating income is P22.000 12.20 x 5/7) 0.500 B.500 + P22.500 C.00 P3. the composite BEP in units shall be: Composite BEP (pesos) = (FC + IBIT) / Average UCM = (P49.500 D.900 (cia) 70.50 and P1.500 (cia) 71.50 x 2/7) P0.500 units x 5/7) = 32. Fixed manufacturing cost is applied at a rate of P1. 000. and its UCM is P0. Product BD-4 should be produced.50 (i.00 (i. P1. Product Xy-7 should be produced. and its UCM is P1. 640.000 / 16-2/3% = P960.000 units. yielding a contribution margin of P250. If the potential increase in sales units for either product resulting from advertising is far in excess of this production capacity. Product XY-7 should be produced..000 units.000 units.000 units.e. B ? The increase in unit sales to offset the increase in advertising. Suppose the sales manager chooses to devote the entire P160.00). P320. The increase in peso sales to offset the increase in fixed advertising cost is: Increase in peso sales = Increase in fixed cost / CMR = P160.500.000 (cma) 73. . 333. yielding a contribution margin of P133. C. The products are not substitutes for one another in the eyes of the company’s customers. The increased in unit sales to offset the increased in advertising is: Increase in unit sales = Increase in fixed cost / UCM = P160. yielding a contribution margin of P75. Suppose Margarita has only 100.. B.e.000. D.000 74.000 C.000 increase in the money to the most profitable product. 80. P1. P 960. C.  The unit variable cost of BD-4 is P2. which product should be advertised and what is the estimated increase in contribution margin earned? A.600.000 / P1 = 160.00 – P2.. B. P3.Chapter 4 Marginal costing and CVP analysis 120 The sales manager has a P160.000 machine hours that can be made available to produce additional units of XY-7 and BD-4.50 + P1. 72. yielding a contribution margin of P187. Its CMR is 16-2/3 %. D. P4 – P3). 000 B.00 (i. The minimum increase in sales units of XY-7 to offset the increased advertising is A.50). Product BD-4 should be produced. P2 + P1). C ? The increase in peso sales of produce BD-4 to offset the increase in advertising expense.000 D.000 units 73. Suppose the sales manager chooses to devote the entire P160.000 to increased advertising for XY-7.50 (i.  The unit variable cost of XY-7 is P3. 128. 000 to increase advertising for BD-4.e.. the minimum increase in revenues of BD-4 to offset the increased advertising would be A. P160.e. 160. (cma) 72. then the contribution margin ratio will A. Rise D.. It is given that the fixed overhead is applied at P1.  A shift in mix towards the less profitable product will make the average unit contribution margin to decline. Decrease C.75 hr.  The shift in sales mix from the less profitable to the more profitable product would increase the average contribution margin. The total CM in producing BD-4 is P250. Sari. Choiceletter “c” is also incorrect because CMR is affected by the change in sales mix.000 hours x P2. 75.50 ÷ No. D ? The product to be sold and its estimated increase in contribution margin. as the mix of the products being sold changes. the overall contribution margin ratio will also change. accounting for 30% of gross sales. Choice-letter “b” is incorrect because it contradicts the correct analysis.00 per hour. Not change B. Increase B. Fall C.75 per unit / P1 per hour) 0. Choice-letter “d” is incorrect because the change in BEP is not directly proportional to the change in CMR.50 Product BD-4 gives a higher CM per hour and should be prioritized for production and sales.50 per hour). Contribution margin per hour P1.sari Corporation is a multiple-product firm.Chapter 4 Marginal costing and CVP analysis 121 (cma) 74. they decided to shift the sales mix from less profitable products to more profitable products. If the shift in mix is toward the less profitable products. In a multi-product company. This will cause the company’s breakeven profit to A. This will result to a lower CMR (choice-letter “a” is correct).e.33 P2. 76. Not change (rpcpa) 76. 100.000 (i. This means a decrease in the composite .00 P0.20 hr.20 per unit / P1 per hour) 0. A ? The effect to contribution margin ratio (CMR) if the sales mix is shifted towards the less profitable product. D. A ? The effect on the breakeven point (BEP) if the sales mix is shifted from less profitable products to more profitable products. of hours per unit (P. The contribution margin per hour shall be computed as follows: XY-7 BD-4 UCM (as computed in the preceding number) P1. In their review of operations. Change by 15%.  The unit contribution margin of each product must be converted into contribution margin per hour to determine which is more profitable. Change in direct proportion to break-even point (rpcpa) 75. (P. 00) Unit direct labor cost ( 15.00 (15.  Composite BEP refers to the point where the total sales of all the products made by the company is equal to the total costs.00 (10.50 Composite BEP (units) = Fixed costs / Average UCM = P4.00 P 35.25 P 26.00 P 75.000 units B. Phil.00) Direct labor (15. In numerical calculations.875.545 units D.000 units C. 150. Frames Inc. Assume that the company plans to maintain the proportional mix.00) P15.00 100.00) (15.00 Direct materials (10. has the following revenue and cost budgets for the two product it sells Plastic Frames Glass Frames Sales price Direct materials Direct labor Fixed overhead Net income per unit Budgeted units sales P50.00) (20. the company rounds to the nearest centavo and unit. 354.000. sells are made available: Plastic Things Glass Things Sales price P50. based on the given budgeted unit sales.00) Unit contribution margin P 25.000 / P32. The following revenues and cost budget for two products Things Inc. The fixed costs is given at P4. 75.000.00) (15. 300.000 units (rpcpa) 77. Choice-letter “b” is not determinable and is incorrect.25 P32..00) .000 The budgeted unit sales equal the current unit demand.000 P75.00 Unit direct materials cost ( 10. respectively.00) ( 25. and total fixed overhead for the year in budgeted at P4. and costs structure. The composite BEP is equal to fixed costs divide by average unit contribution margin (UCM).Chapter 4 Marginal costing and CVP analysis 122 BEP (choice-letter “a” is correct).00) P10.875.00) ( 15.00) (15.000 P75. Choice-letter “c” is incorrect because it contradicts the results of the correct analysis.00 300.00) (25.00 X Sales mix ratio (1:3) ¼ ¾ Adjusted UCM P 6. 77. These products made by the company is equal to the total process.000 units 78. Choice-letter “d” is incorrect because there is a change in the breakeven point due to the change in sales mix. C ? The composite breakeven point.50 = 150. The average UCM shall be computed using the sales mix ratio of 1:3 for plastic and glass. The total number of units the company needs to produce and sell to break-even is A.875. Plastic Glass Average UCM Unit sales price P 50.00) (25. At which sale increases.947 units.Chapter 4 Fixed overhead Net income per unit Budgeted units sales Marginal costing and CVP analysis (15.. The diagonal line is the contribution margin line. Choice-letter “b” is correct. At which the total costs equal the total sales.00 Unit variable costs (25. (rpcpa) 79. In numerical calculations. B. total costs equal the total sales.000.632 units. 15:30) is taken from the relationship of the budgeted units of 150.00 300.000 123 The budgeted unit sales equal the current unit demand.) The composite BEP is computed by dividing the total fixed costs and expenses by the average unit contribution margin (UCM).00) (40.00 P75.00 150. (rpcpa) 78. In a profit-volume graph. 171.000 units and 300. The point at which the contribution margin line intersects the zero profit line is the point: At which the volume level is zero. Choice-letter “a” is incorrect because there is a sale made and therefore the volume level is higher than zero. At this point. Choice-letter “c” is incorrect because a change in sale would affect the amount of contribution margin and not at point zero. needs to produce and sell to break-even is A. The average UCM is determined below: Plastic Glass Average UCM Unit sales price P50.000 (20. respectively. 418.33 P23.00) P15.00) P10. then there is no profit and the business operates at the breakeven point. B ? The point at which the contribution margin line intersects the zero profit line in the profit-volume graph.g.e. The vertical axis is the profit in pesos and the horizontal axis is the volume in units.  If the contribution margin line intersects the zero profit point. At which total variable costs equal total sales. C. D.00 X Sales mix ratio (15:30) 15/45 30/45 Adjusted UCM P 8.. 153. and total fixed overhead for the year is budgeted at P4. The total number of units Thing Inc.947 units The sales mix ratio (i.000 units for plastic and glass products.875. B.33 P31. 102. C.66 Composite BEP (units) = P4. D.66 = 153.875.000 / P 31. Assume that the company plans to maintain the same proportional mix. . the company rounds to the nearest centavos and unit. B ? The composite breakeven point.958 units.00 35.455 units. the cost/volume/profit relationships are represented. Choice- A. total sales equal total costs.00) Unit contribution margin 25. Breakeven and CVP graphs 79..  Composite breakeven point is the total or combined sales in units of all products produced and processed in the same plant in order to breakeven (e. there is already a margin of safety and profit because total sales already exceed total costs and total contribution margin is greater than fixed cost and expenses (choice-letter “c” is correct). choice-letter “b” is incorrect. There is loss because the total cost line exceeds the total revenue line. and not to exceed. C. Hence. 80. will result to profit. 81. cost.00 P 4. Expenses are extremely high relative to revenues. Total sales exceed total cost. C. and therefore.25 Budgeted unit sales 100. B. D. choice-letter “a” is incorrect.75) Net income per unit P 2.  After the breakeven point. D. When using the graph method.00) (3. revenues. C ? The most important use of the CVP graph. There is profit since the total cost line exceeds the total revenue line. (rpcpa) 80. (rpcpa) 81.000 300. The relationships among volume. MultiFrame Company has the following revenue and cost budgets for the two products it sells: Plastic Frames Glass Frames Sales price P10. Sensitivity analysis Questions 82 through 84 are based on the following in formation. The most important use of the cost-volume-profit graph is to show A. The determination of cross over point. C ? Using the graph method. then there is no contribution margin. the total cost line should be exceeded by. A.000 . Choice-letter “d” is incorrect because if there is profit.00) Fixed overhead (3.00) Direct Labor (3. if unit output exceeds the break-even point. The breakeven point is not the main focus of presentation but its focus is the effects and relationships to profit of revenues. over wide ranges of activity. Choice-letters “a” and “b” are incorrect because after breakeven point revenues are already higher than expenses. costs and volume within a given relevant range.Chapter 4 Marginal costing and CVP analysis 124 letter “d” is also incorrect because if total variable costs and expenses equal total sales. The breakeven point. Choice-letter “d” is an irrelevant choice.00 P 15. The cost/margin ratio at various levels of sale activity. the event that will happen when unit output exceeds the breakeven point.00 Direct materials (2.00) (5.  The CVP graph and even the BEP graph show the relationships among revenues.00) (2. costs and volume. the total revenue line. B. The contribution margin ratio is assumed to be constant over the relevant range. 000 units of glass frames with all other costs remaining constant is A.Chapter 4 Marginal costing and CVP analysis 125 The budgeted unit sales equal the current unit demand. 153.75 The composite BEP shall be 144.. the new average UCM shall be: Plastic frames (P6 x ¼) P1. 100. This decrease in unit variable cost will increase the UCM of plastic frames to P6 (i. C. 300. (cma) 83.25 5.50 The composite BEP shall be 150. C. P5 + P1). B. 144. In numerical calculation. A ? The new composite BEP assuming a change in direct labor cost.. The average UCM is determined as follows: Plastic (P10 – P5) Glass (P15 – P8) Total Sales UCM mix ratio P5. The total number of units needed to break even If the budgeted direct labor cost were P2 for plastic frames instead of P3 is A. .947 units.000 units of plastic frames and 300.000 units. The total number of units MultiFrame needs to produce and sell to break even is A. (cma) 82.444 units (i. D. 144.e.  The requirement refers to the composite breakeven point. 150. B.000 / P6. MultiFrame rounds to the nearest cent and unit 82.000 / P6.000 units. respectively.75).000 units. 83. and total fixed overhead for the year is budgeted at P975. 150.50 Glass frames (P7 x ¾) 5. C.  The direct labor cost of plastic frames decreases to P2 from a previous balance of P3 or a decrease of P1. The total number of units needed to break even if sales were budgeted at 150. 84.50). With this change. 153.00 ¾ Ave.00 ¼ 7.000 units. 100. It is calculated as fixed costs over average UCM.000 units. 153.25 P 6.947 units. The sales mix shall be based on budgeted sales (1:3) and the sales mix ratios are ¼ and ¾ for Plastic and Glass. A ? The total number of units needed to produce and sell to breakeven.444 units.444 units. UCM P1.e.947 units.25 Average UCM P6. Assume that the company plans to maintain the same proportional mix. P975. D.000 units (P975.000. we will have a new BEP in units of 51. study shows that with the use of the new equipment sales volume in units are expected to increase by 40% while variable manufacturing costs will decrease from P2. 450.34 or P6. The company has been operating at full capacity. The total fixed selling and administrative expenses and variable selling and administrative expenses will remain the same.e. P975. (cma) D.. it was decided by the management committee to lower the price to P36 without sacrificing the quality of the product.000 450. C ? New composite BEP given a change in sales mix. The selling price per unit will also remain the same. .33333 The new composite BEP shall be 153.000 ÷ P6.625 C. 43. Calculate the overall breakeven point in terms of units if the company believes that the current price of P40 is too high and the firm faces stiff competition.  The UCM of plastic frames and glass remains constant at P5 for plastic frames and P7 for glass frames.67 Average UCM P6. respectively..250 B. Marginal costing and CVP analysis 150. 39.947 units (i. 126 84. The new sales mix is changed to 15:30 for plastic frames and glass frame.000 The company is concerned about the expected increase in fixed manufacturing costs by 50% if it will buy a new equipment with a higher production capacity.000 units) Cost of goods sold: Fixed Variable Gross profit Selling and administrative: Fixed Variable Income before taxes P 600.000 39. A ? The new breakeven point in units.50 per unit. 85. 31.000 P 66.67 Glass frames (P7 x 30/45) 4.Chapter 4 B. The new average UCM shall be: Plastic frames (P5 x 15/45) P1.000 units.333333)..000 P 150. After all the sensitivity analysis is done. 625 units (i.000 units.000 84.000 45. Presented below are the results of operations of Softtouch Products. Questions 86 and 87 are based on the following information. If the company will buy the new equipment.750 and unit contribution margin is P6? A. 51.000 150. Inc.e. for 2005: Sales (150.  The new breakeven point is equal to new fixed costs divided by the new unit contribution margin. What is the new breakeven point if fixed costs are P309.750 (rpcpa) 85.750 / P6). Applying this.000 300. However.125 D.00 to P1. P309. 000x150%) Total fixed expenses 39.  The changes in the data could be analyzed as follows: Before After Unit sales price (P600. Therefore.000 B.000 .00 B. A ? The maximum expected income before income tax.000 (rpcpa) 86.000 Total fixed costs and expenses P189. Data for the previous year were as follows: Selling price per piece P 8.000 39.50 (rpcpa) .000 P225.00 P 4.000 Income before income tax P198.000 (P150.850 For the coming year.000) ( 2. the company estimates that the selling price will be P9. P 8. P 288.000 C. P 9. What would be the breakeven point in terms of units? A.000 / P2. Income tax rate of 35% will not change. and fixed costs will increase by 10%.Fixed costs and expenses 264. P306.20) P462. 66.000 (rpcpa) 87.000 B.00) ( 1.000 Questions 88 and 99 are based on the following information.000 87.000 / 150.00 C. 128.00 Unit variable production costs (P300.Chapter 4 Marginal costing and CVP analysis 127 86.000 The new breakeven point in units is (P264.000.00 No.  Income before income tax is contribution margin less fixed costs and expenses. 176.000 / 150. A ? The new breakeven point if the company buys the new equipment. of pieces to breakeven 25.20 120. P 216.50) Unit variable expenses (P45. Variable cost to manufacture will increase by 25%.000 D. 105.50 per piece.50 D. What is the selling price per piece that would give the same contribution margin rate as previous year? A.00 Variable cost per piece P 2. 88. What is the maximum expected income before income tax? A. the maximum income before income tax is P198.20 Total fixed manufacturing cost P150. P10.30) ( 0. calculated as: Contribution margin (150.000 D.000 P264. The Kabayan Company manufactures and sells Batik handbags to assorted prints.000 units x 140% x P2.000) P 4.000 C. P10.000) ( 0.30) Unit contribution margin P 1. P198.70 P 2.000 Net income last year P5.000 / 150. 000 B..e. 26..000 / Unit sales price P 8. 28. The unit variable cost is P6 (i. C ? Operating income given some changes in the variables of profit. The operating income in 2006 is calculated as follows: Sales [(100. 26. (rpcpa) 89.. P2.500 . P 190.e.850 / 65%) 9.000 were spent on advertising. P600.000 / Contribution margin ratio 75 % Sales 212.800 units.00 Sales volume last year 26.e.00 x 125%).800 B. A ? The selling price per unit that would give the same CMR as the previous year.300 units 90.50 (i.000 x P6) P150.000 Contribution margin 159.300 C.000 / 100. 89. what would be the expected sales volume for the coming year? A. P 175.800 pieces. Wilson Company prepared the following preliminary forecast concerning product G for 2006 assuming no expenditure for advertising: Selling price per unit P 10 Units sales 100. A ? The expected sales volume in the coming year. 27.265.50 / 25%).000 units x 110%) x (P10 x 115%)] P1.000 D. Assuming that Wilson incorporates these changes in its 2006 forecast.  Profit is sales less costs and expenses.000 Income before income tax (P5. P 365.000 Variable costs P 600.e. If sales for the coming year are expected to exceed last year’s by 1. If the new unit variable cost is P2. Wilson estimated that it could increase the unit selling price by 15% and increase the unit sales volume by 10% if P100.Chapter 4 Marginal costing and CVP analysis 128 88. what should be the operating income from product G? A. then the new unit sales price to maintain the same CMR of 75% and the same VCRatio of 25% shall be P10.  The old variable cost ratio is 25% (i. P2.000 .000 units).000 (aicpa) 90.000 C.  The sales volume in the coming year shall be the sum of the sales volume last year plus the expected increase of 1.500 units Add: Increase in sales volume this year 1. P 205.000 Based on market study in December 2005. 100% – 25%).000 Fixed costs P 300.775 D.800 Expected sales in units in the coming year 28. P2 / P8) and the CMRatio is 75% (i.00 (i. The sales volume last year and the new sales volume is determined as follows: Fixed costs (25.e. Computes profit per unit of production and determines the optimum production of the company.000 units at P30) P4. D. B.Chapter 4 Marginal costing and CVP analysis 660. Allows the decision maker to introduce probabilities in the evaluation of decision alternatives.000) 6 Operating income P 450.500. and the sales price would remain as they were in 2005.000 10 Mfg. Rainbow Company’s controller developed the following variable-costing income statement for 2006: Per Unit Revenues (150. Applied in the CVP analysis. (rpcpa) 91.000 9 Fixed costs: Mfg. but with operating income being reduced to P300.000) Operating income 91. Determines the most profitable mix of product to be sold.Variable costs [(100. Choice-letter “c” is incorrect because it also refers to linear programming.000 400. C.000 P3 Rainbow Company based its 2006 budget on the assumption that fixed costs.050.000 2 ( 900.000 2 ( 3.Fixed costs (P300.150.000 2 Selling & marketing 300. it refers to the process of determining the effects to profits of the possible changes in the variables affecting profit (choice-letter “d” is correct).000 129 .000 + P100. Choice-letter “b” is incorrect because it refers to making decisions under uncertainty where the concept of probabilities is applied.  Sensitivity analysis (or simulation analysis) is the process of determining the effects of the possible changes in any of the variables to the overall outcome in a given circumstance. D ? The statement describing sensitivity analysis as used in cost-volume-profit analysis.000 4 Selling & marketing 300.000 units x 110%] x P6] Contribution margin .350. overhead P 600. Is done through various possible scenarios and computes the impact on profit of various predictions of future events. By July of 2006. Choice-letter “a” is incorrect because it refers to linear programming where the optimum mix of production and sales is computed to maximize profit.000) 21 Contribution margin 1.000 P30 Variable costs: Direct materials P1.500. the controller was able to . 92. Overhead 300.000 P 205. 000.000 7 Direct labor 1. When used in cost-volume-profit analysis.000 605. unit sales. sensitivity analysis A. 000 Projected CM 1. which includes P60.000 D.00 The projected operating income is revised on July 1. as follows: CM (150. 2006 when volume is expected to increase by 10% over that of the 2005 level.200. The maximum after-tax income that can be earned by Levi Company from sales of the new product during the next fiscal year is A.00 Total variable manufacturing cost 14. and profit decreases to P300. 000 for depreciation on new manufacturing equipment.00 Direct labor 3. if units sold remain constant.000.50 Total variable cost P16.00 93.  Operating income is sales less total costs and expenses. P110. Levi is subject to a 40% income tax rate. units sold increases by 10%.000 .000 Questions 93 and 94 are based on the following information. A . P 66.000 x 110% x P8) P1. C ? The predicted operating income in 2006.Fixed costs 900.000 Projected operating income P 420. Based on the 2006 budget and the new information.000 C. 2006 is: Projected operating income P 300. P330.000 units could be sold at P36 per unit.000 B. P585. Levi Company has developed a new project that will be marketed for the first time during the next fiscal year. P420.000 / Units sold (assumed unchanged) 150.50 Selling expenses 1. P30.000 units of the new product annually.50 Manufacturing overhead 4.000 + Fixed costs 900. P50. The fixed costs associated with the new product are budgeted at P450. Data associated with each unit of product are presented below.000 B. the predicted 2006 operating income would be A.Chapter 4 Marginal costing and CVP analysis 130 predict that unit sales would increase over 2005 levels by 10%. It is predicted that total fixed costs and unit sales price are unchanged.320.000 (cia) 92. The UCM as predicted on January 1.000 (cma) 93.000 for the year.000 D.000 units UCM if units were unchanged P 8. Variable Cost Direct material P 7. P300. Although the Marketing Department estimates that 35.000 C. Levi management has allocated only enough manufacturing capacity to produce a maximum of 25. 000 Fixed costs 450. B ? Unit sales price if operating income is P200. David’s 2006 liability insurance increased by P1.Income tax (40%) 20. At a volume of 80.000 units. Assuming the volume and other costs are unchanged.000 .000 Contribution margin P575.  Unit sales price is unit variable cost plus unit contribution margin.000 Net income P 30. P36.2 million. P 150. computed below: IBIT [P75.000 / (1-. while total fixed costs increases to P2.00 (aicpa) 95.00 B.00 39. P34.Fixed costs 450.. 000 the first year.00 16.000 Income before income tax 50. David had fixed costs of P 1 million and operating income before income taxes of P200.00 C.000 .200. The unit-selling price to achieve this target income must be at least.Chapter 4 Marginal costing and CVP analysis 131 ? The maximum after-tax income of the new product next year. P39.e.  Unit sales price is unit variable cost plus unit contribution margin. Because of an adverse legal decision.200.000 / 25.000 P 200.000 2.000 (P1 million + P1.000. A.000 94.000. The UCM in 2006 shall be determined as follows: 2005 2006 Income before income tax P 200.2 million) Contribution margin 1. what should the 2006 price be if Thor is to make the same P200.00 C. The net income is computed as follows: Contribution margin (25.000 Fixed costs 1. P 135.60 D.000. P 120.00 D.000 operating income before income taxes? A. D ? The targeted unit sales price.40)] P125. During 2005. P36 – P16). P 240. P37.00 The UCM is 95.  The management has decided to produce and sell only 25.400.000 2.000 units x P20) P500. The UCM of the product is P20 (i. Levi Company’s management has stipulated that it will not approve the continued manufacture of the new product after the nest fiscal year unless the after-tax income is at least P75.00 (cma) 94. The unit variable cost last year does not change.2 million over 2005.000 Unit contribution margin (P575.60 B.000 units) P Unit variable cost Unit sales price P 23. David Lab supplied hospital with an comprehensive diagnostic kit for P120.000 .000 kits. 574. P 280.Chapter 4 Marginal costing and CVP analysis 80.320.000 D.Breakeven sales (P2. P 150.000 . Proportionate change in net income. P 4. the annual revenues would not be the projected P6 million but would have to A.  The variable cost ratio is 45% (i. . P 7.850. P 82. the wholesale distributing company’s margin of safety in terms of revenues for 2006 would be A. 30% + 5% + 10%) and CMR is automatically 55%. P 5.000.320. If the wholesale distributing company wants to earn the same before-tax operating income in 2006 as budgeted for 2005.000 B. P 5.500) / 55% = P5.000 calculated as follows: Sales (pesos) = (FC + IBIT) / CMR = (P2. that is.800). Bi Corporation is operationally.801. The company is preparing its annual budget for 2006 and has accumulated the following data: Projected annual revenues P6.  Margin of safety is budgeted sales less breakeven sales.000.000 / 55%) 4.800 96.000 (cia) 97. Using the original P 6 million projection.000 Margin of safety P1.574.362.000 30 105 135 132 / Units sold in 2004 80.000 B.500 C.500 for 2005.e.000 C.000 (cia) 96. Total fixed costs is P2.000 Operating leverage 98. it has high fixed costs and low variable costs. computed as follows: Budgeted sales P6.200 + P1.000 (i. A wholesale distributing company has budgeted its before-tax profit to be P643.680.950.200 Administrative expenses P1. If the income in 2005 is retained in 2006.000 97. C ? Projected sales in 2006. a highly leveraged company. the needed sales in 2005 shall be P5.574. D ? Margin of safety in 2006.801.850.. P772.000 + P643. P1.850.150. small changes in sales volume result in A.000 Variable costs as a percent of revenues: Cost of merchandise 30% Sales commissions 5% Shipping expenses 10% Annual fixed operating costs: Selling expenses P 772.000 Unit contribution margin P 15 Unit variable cost (P120 – P15) P 105 Unit sales price P P P Questions 96 and 97 are based on the following information.e. As such.000 D.. DOL = CM / EBIT Also. Operating leverage. (cia) 99.Two companies are expected to have annual sales of 1 million decks of playing cards next year. B.30 .000 Given these data. This is called the degree of operating leverage (DOL). Negligible change in net income. which of the following responses is correct? Volume in Units Volume in Units . Financial leverage. Marginal costing and CVP analysis 133 Large changes in net income.62 . The higher the DOL. a change in sales volume does not result to a proportionate change in net income. the higher the change in EBIT in times of increasing trend in sales. C.00 P3. Combined leverage. Breakeven leverage.  A company with a high degree of operating leverage would also result to a high degree of change in EBIT (earnings before income tax) in every small change in net sales. D. The DOL computations are presented below: That is. The percentage change in earning before interest and taxes associated with the percentage change in revenues is the degree of A. Another mode of determining the DOL is dividing contribution margin by the operating income. Therefore.Chapter 4 B. D.13 . Choice-letters “c” and “d” are also incorrect because the percentage change in net income is directly affected by the degree of operating leverage. Estimates for next year are presented below: Company 1 Company 2 Selling price per deck P3. A ? Percentage change in EBIT with the percentage change in revenues  The percentage change in EBIT in relation with the percentage change in sales measures the elasticity of change in profit with the change in sales. DOL = Percentage change in EBIT x Percentage change in sales Indifference point 100.00 Cost of paper per deck . No change in net income. C.65 Printing ink per deck .25 Variable overhead per deck .35 Fixed costs P960.75 1.000 P252. (rpcpa) 98. B ? Effect of a high degree of operating leverage if there are small changes in sales volume. This relationship is expressed in the formula: ٪Δ in EBIT = ٪Δ in Net Sales x Degree of Operating Leverage Choice-letter “a” is incorrect because the percentage change in net income is not only affected by the degree of operating leverage but that of the percentage change in net sales.15 Labor per deck . 99. It has an investment of P1.  The BEP for each company shall be: Company 1 Company 2 Unit sales price P 3.25 B.000 1.000/2.00 Unit fixed expenses (P150.500 per table.180. 800.75 . P2.20x – P960.000 for selling and administrative expenses.20 0.62 + P.750. The profits of the two companies are expressed as follows (assume “x” as the number of units sold): P1 = 1.000 Breakeven Breakeven at which profits Point in Point in of Company 1 Units for Units for and Company 2 Company 1 Company 2 Are Equal C. 000 = 0. 1 P2 = 0.00 (rpcpa) 101.2 At indifference point: Therefore: P1 = P2 1. manufactures computers tables.000 1.000 420.30) 1.60 BEP in units (P960.531.60x – P252. P2.00 Unit fixed cost P550.60) The indifference point is where the results or profits between the alternatives would be the same.500.75 C.00 Unit variable cost (P.000.000) 275.65 + P.000 x = 1.20x + P960.00 420.NCB Inc.  Unit sales price is the sum of unit variable costs and expenses. The selling price per table should be: A.180.000 420.80 2.000 x 25%) / 2.750.068.A ? BEP and indifference point.000 B.35) Unit contribution margin 1.Chapter 4 Marginal costing and CVP analysis 134 Breakeven Breakeven at which profits Points in Point in of Company 1 Units for Units for and Company 2 Company 1 Company 2 Are Equal A.75 + P.000 1.A ? The selling price per table. 800.25 + P.000 ÷ P.000 units is P550.000 units Miscellaneous 101.40 (P.334 105.334 105.60x – P252.068.000) 75.75 Unit sales price P2.00 D.000 1.000 P2 = profit of Co. unit fixed costs and expenses.000 ÷ P1.180. The variable cost to manufacture is P1.00 P 3.000 where: P1 = profit of Co.20) 800.13 + P.000 D.725. 533.00 Unit profit margin [(P1. P1.000 plus an additional P150. 533.000 (P252.000 units] 218.000.000 in assets and expects a 25% return on investment. P2. Its fixed production cost for 2.000/2.850.000 (cia) 100.15 + P1. and unit profit margin as follows: Unit variable costs P1. Using absorption costing. does not affect BEP calculation and is the correct answer.  Choice-letter “b”. (rpcpa) 102. The amount of contribution margin is equal to sales multiplied by the CMRatio. The fixed cost is not given but could be determined by getting the difference of contribution margin and profit before tax. The procedural analysis follows: a. The A/R turnover is equal to 360 days in a year divided by the collection period. Choice-letter “c”. 103. Assume a 360-day year. except A. BEP (pesos) = P1. The number of units sold during the period D. and is relevant in determining the breakeven point. P2. Choice-letter “d”. achieving targeted production levels.000 Fixed costs P1. budgeted level of production.000 103.000 x A/Receivable Turnover (360 days/15 days) 24 times Net sales 3.106.000.040 b. is also important in determining the contribution margin and the BEP. the number of units sold. P3. Determine the company’s breakeven revenue.204 . The ending receivables balance is the average balance during the year.B ? The item that is not included in determining the breakeven point.096. P2. Average Accounts Receivable P 129.023. the determination of the break-even point depends on all of the following.A ? Breakeven revenue.000 units of “yo” a top-of-the-line garden srinkle.106. Achieving targeted production levels C. is definitely relevant in determining the BEP.Chapter 4 Marginal costing and CVP analysis 135 102. The CMR is given but the amount of net sales is to be computed using the receivable turnover analysis.608 B.040 Less: Profit before tax 485.000 X CMRatio 49% Contribution margin 1.040 / 49% = P2. Net sales equal average accounts receivable balance times the receivable turnover.032. Choice-letter “a”.  Breakeven revenue or breakeven peso sales is equal to fixed cost over CMRatio.Hello Garci Company sells 50.032. All sales are on credit.122 C. the level of fixed manufacturing overhead. is the basis in computing total fixed costs and unit variable costs.000. • Days of sales outstanding 15 days. • Contribution margin ratio 49%. These were taken from the company’s records: • Accounts receivable P129. The budgeted level of production B.517.096. P1.032.040 D. The level of fixed manufacturing overhead. • Profit for the period was P485. A. Statement 2 is correct because a higher contribution margin ratio would result to higher profit for every peso of sale after the breakeven sales. True False True True True True C. Statement 1 Statement 2 Statement 3 Statement 1 Statement 2 Statement 3 A.  Statement 1 is false because the sum of variable expenses and fixed expenses is the total expenses. D. Statement 1-The breakeven point is defined as the sum of variable expenses and fixed expenses. Statement 3-All fixed costs are treated as period costs when variable costing is used. Hence. Statement 3 is also true because fixed costs and expenses are considered as period costs (automatically deducted from revenues) under the variable costing system.Chapter 4 Marginal costing and CVP analysis 136 104. choice-letter “b “ is the correct answer. B.B ? Identify the statements as true or false. Statement 2-As sales exceed the breakeven point. False False False False True False (rpcpa) 104.State whether the following statements are true or false. and not the breakeven point. a low contribution margin percentage would result in lower profit than would a high contribution margin percentages.  done  .
Copyright © 2024 DOKUMEN.SITE Inc.